Physics
Physics, 23.03.2021 01:40, keke6361

If the bag weighs 50 N and your bicep connects to your arm 2 cm from the pivot point, how much force must your bicep exert in Trial 1 (with the bag in your hand)?

answer
Answers: 3

Other questions on the subject: Physics

image
Physics, 22.06.2019 05:50, lola5683
Sawyer is studying diffraction. he draws a diagram of a plane wave to show how light waves travel. which best describes sawyer’s error? the wave fronts should be perpendicular to the direction in which the waves move. the arrow showing the direction of movement of the waves should be pointing to the left. the arrow showing the direction of movement of the waves should be pointing downward. the wave fronts should be both parallel and perpendicular to the direction in which the waves move.
Answers: 2
image
Physics, 22.06.2019 06:40, danny1687
Use the right-hand rule for magnetic force to determine the charge on the moving particle. this is a charge.
Answers: 1
image
Physics, 22.06.2019 12:50, natalyarenassalgado
Arunner is jogging at a steady 3.6 km/hr. when the runner is 2.9 km from the finish line, a bird begins flying from the runner to the finish line at 14.4 km/hr (4 times as fast as the runner). when the bird reaches the finish line, it turns around and flies back to the runner. even though the bird is a dodo, we will assume that it occupies only one point in space, i. e., a zero-length bird. how far does the bird travel? (b) after this first encounter, the bird then turns around and flies from the runner back to the finish line, turns around again and flies back to the runner. the bird repeats the back and forth trips until the runner reaches the finish line. how far does the bird travel from the beginning? (i. e. include the distance traveled to the first encounter)
Answers: 2
image
Physics, 22.06.2019 13:50, cathydaves
Most of the volume of an atom is occupied by the
Answers: 1
Do you know the correct answer?
If the bag weighs 50 N and your bicep connects to your arm 2 cm from the pivot point, how much force...

Questions in other subjects: